Diễn Đàn MathScopeDiễn Đàn MathScope
  Diễn Đàn MathScope
Ghi Danh Hỏi/Ðáp Thành Viên Social Groups Lịch Ðánh Dấu Ðã Ðọc

Go Back   Diễn Đàn MathScope > Sơ Cấp > Đại Số và Lượng Giác

News & Announcements

Ngoài một số quy định đã được nêu trong phần Quy định của Ghi Danh , mọi người tranh thủ bỏ ra 5 phút để đọc thêm một số Quy định sau để khỏi bị treo nick ở MathScope nhé !

* Nội quy MathScope.Org

* Một số quy định chung !

* Quy định về việc viết bài trong diễn đàn MathScope

* Nếu bạn muốn gia nhập đội ngũ BQT thì vui lòng tham gia tại đây

* Những câu hỏi thường gặp

* Về việc viết bài trong Box Đại học và Sau đại học


Trả lời Gởi Ðề Tài Mới
 
Ðiều Chỉnh Xếp Bài
Old 19-07-2012, 06:33 PM   #16
Ispectorgadget
+Thành Viên+
 
Tham gia ngày: Dec 2011
Đến từ: Hồ Chí Minh city
Bài gởi: 98
Thanks: 53
Thanked 126 Times in 57 Posts
Bài 9: CMR nếu $a,b\in (0;1]$ thì $$a^{b-a}+b^{a-b}\leq 2$$
[RIGHT][I][B]Nguồn: MathScope.ORG[/B][/I][/RIGHT]
 
__________________
$F\begin{Bmatrix}
\heartsuit
\end{Bmatrix}=\frac{1}{\sqrt{2\pi}}\int_{-\infty }^{+\infty }f(t)e^{it\heartsuit}dt=? $
Ispectorgadget is offline   Trả Lời Với Trích Dẫn
Old 22-07-2012, 08:09 AM   #17
ptk_1411
Moderator
 
ptk_1411's Avatar
 
Tham gia ngày: Apr 2011
Bài gởi: 698
Thanks: 162
Thanked 813 Times in 365 Posts
Trích:
Nguyên văn bởi Vinh Phuc View Post
2. Không post chen ngang:

KHÔNG Post lời giải bài cũ trước khi post đề mới.
Các bạn hết sức lưu ý quy định 2. Một ví dụ cho vi phạm này là Bài 10, nhưng vì đã có bạn giải nên mình không muốn xóa. Bắt đầu từ bây giờ, tất cả các post đề vi phạm (cho dù đã được giải) sẽ bị xóa (và tất nhiên, post giải cũng sẽ bị xóa). Mong các bạn chấp hành, vì topic mới lập nhưng đã có chiều hướng lộn xộn .

Trong topic hiện còn 2 bài tồn đọng, nên các bạn không post đề mới nhé

Trích:
Nguyên văn bởi zớt View Post

Bài 6 : Cho $a,b \in \mathbb{R} $. Tìm Max :
$ \frac{a+b}{(a^4+4)(b^4+4)} $
Trích:
Nguyên văn bởi Ispectorgadget View Post
Bài 9: CMR nếu $a,b\in (0;1]$ thì $$a^{b-a}+b^{a-b}\leq 2$$

[RIGHT][I][B]Nguồn: MathScope.ORG[/B][/I][/RIGHT]
 
__________________
P.T.K
Có xa xôi mấy mà tình xa xôi...
ptk_1411 is offline   Trả Lời Với Trích Dẫn
Old 22-07-2012, 10:59 AM   #18
Lê Đình Mẫn
Moderator
 
Tham gia ngày: Mar 2012
Đến từ: Quảng Bình
Bài gởi: 19
Thanks: 17
Thanked 15 Times in 9 Posts
Trích:
Nguyên văn bởi zớt View Post
$a^3+b^3+c^3\leq 1+ab+bc+ca $
vì $a^2+b^2+c^2=1 $ nên
$ a^2+b^2+c^2+ab+bc+ca \geq 0 $ hay
$ 1+ab+bc+ca \geq 0 $
TH1: nếu $ a^3+b^3+c^3<0 $=> Đpcm.
TH2: Nếu $ a^3+b^3+c^3 \geq 0 $
Vì bây giờ cả 2 vế BDt không âm, bình phương 2 vế ta được:
$ a^2b^2+b^2c^2+c^2a^2+(a+b+c)^2 + \sum 2a^bc \geq \sum a^6 +\sum 2a^3b^3 $

$\Leftrightarrow \sum a^2b^2+ \sum 2ab+ \sum a^2 +\sum 2a^bc \geq \sum a^6 +\sum 2a^3b^3 $

$\Leftrightarrow \sum a^2b^2 +\sum 2ab+ (\sum a^2)^2 + \sum 2a^bc \geq \sum a^6 +\sum 2a^3b^3 $

$\Leftrightarrow \sum 2a^2b^2 +\sum a^2b^2 + \sum 2ab + \sum a^4 + \sum 2a^2bc \geq \sum a^6 +\sum 2a^3b^3 $

Do $ a^2+b^2+c^2=1 \Rightarrow (a,b,c) \in [-1,1] $
Ta cần Cm $ \sum 2a^2bc \geq 0 \Leftrightarrow \sum \frac{1}{ab} \geq 0 $ ÁP dụng BDt schwart ta có đpcm
Từ đó suy ra điều cần chứng minh. Dấu = xảy ra khi 2 số =0,1 số =1
Bài của bạn hình như có vấn đề đoạn sau. Bởi vì $\sum ab $ chưa hẳn dương.
[RIGHT][I][B]Nguồn: MathScope.ORG[/B][/I][/RIGHT]
 
Lê Đình Mẫn is offline   Trả Lời Với Trích Dẫn
The Following User Says Thank You to Lê Đình Mẫn For This Useful Post:
00000 (23-07-2012)
Old 22-07-2012, 02:00 PM   #19
ptk_1411
Moderator
 
ptk_1411's Avatar
 
Tham gia ngày: Apr 2011
Bài gởi: 698
Thanks: 162
Thanked 813 Times in 365 Posts
Trích:
Nguyên văn bởi zớt View Post
Bài 6 : Cho $a,b \in \mathbb{R} $. Tìm Max :
$ P=\frac{a+b}{(a^4+4)(b^4+4)} $
Đặt $k=\sqrt [4] {\dfrac{4}{7}}$. Theo bất đẳng thức AM-GM:

$$\begin{aligned} (a^4+4)(b^4+4)&=a^4b^4+4(a^4+b^4)+16\\&=(a^4b^4+k^ 8)+4(a^4+b^4)+16-k^8\\&\ge 2k^4.a^2b^2+4(a^4+b^4)+16-k^8\\&=k^4(a^2+b^2)^2+(4-k^4)(a^4+b^4)+16-k^8\\&\ge \dfrac{k^4}{4}.(a+b)^4+\dfrac{4-k^4}{8}.(a+b)^4+16-k^8\\&=\dfrac{4+k^4}{8}.(a+b)^4+16-k^8\\&=\left[\dfrac{4+k^4}{8}.(a+b)^4+\dfrac{4+k^4}{8}.(2k)^4+ \dfrac{4+k^4}{8}.(2k)^4+ \dfrac{4+k^4}{8}.(2k)^4\right]+\left[16-3.\dfrac{4+k^4}{4}.(2k)^4-k^8\right]\\&=\left[\dfrac{4+k^4}{8}.(a+b)^4+\dfrac{4+k^4}{8}.(2k)^4+ \dfrac{4+k^4}{8}.(2k)^4+ \dfrac{4+k^4}{8}.(2k)^4\right]\\&\ge 4(a+b).\dfrac{4+k^4}{8}.(2k)^3\\&=(a+b).4k^3(k^4+4 ) \end{aligned}$$

Suy ra $\max P=\dfrac{1}{4k^3(k^4+4)}$, đạt được khi $a=b=k$.
[RIGHT][I][B]Nguồn: MathScope.ORG[/B][/I][/RIGHT]
 
__________________
P.T.K
Có xa xôi mấy mà tình xa xôi...
ptk_1411 is offline   Trả Lời Với Trích Dẫn
Old 22-07-2012, 03:32 PM   #20
JokerNVT
+Thành Viên Danh Dự+
 
JokerNVT's Avatar
 
Tham gia ngày: Dec 2011
Đến từ: Trần Đại Nghĩa high school
Bài gởi: 571
Thanks: 206
Thanked 355 Times in 241 Posts
Trích:
Nguyên văn bởi ptk_1411 View Post
Đặt $k=\sqrt [4] {\dfrac{4}{7}}$. Theo bất đẳng thức AM-GM:

$$\begin{aligned} (a^4+4)(b^4+4)&=a^4b^4+4(a^4+b^4)+16\\&=(a^4b^4+k^ 8)+4(a^4+b^4)+16-k^8\\&\ge 2k^4.a^2b^2+4(a^4+b^4)+16-k^8\\&=k^4(a^2+b^2)^2+(4-k^4)(a^4+b^4)+16-k^8\\&\ge \dfrac{k^4}{4}.(a+b)^4+\dfrac{4-k^4}{8}.(a+b)^4+16-k^8\\&=\dfrac{4+k^4}{8}.(a+b)^4+16-k^8\\&=\left[\dfrac{4+k^4}{8}.(a+b)^4+\dfrac{4+k^4}{8}.(2k)^4+ \dfrac{4+k^4}{8}.(2k)^4+ \dfrac{4+k^4}{8}.(2k)^4\right]+\left[16-3.\dfrac{4+k^4}{4}.(2k)^4-k^8\right]\\&=\left[\dfrac{4+k^4}{8}.(a+b)^4+\dfrac{4+k^4}{8}.(2k)^4+ \dfrac{4+k^4}{8}.(2k)^4+ \dfrac{4+k^4}{8}.(2k)^4\right]\\&\ge 4(a+b).\dfrac{4+k^4}{8}.(2k)^3\\&=(a+b).4k^3(k^4+4 ) \end{aligned}$$

Suy ra $\max P=\dfrac{1}{4k^3(k^4+4)}$, đạt được khi $a=b=k$.
Mình có 1 thắc mắc nhỏ là sao bạn đoán được điểm rơi của bài này là $a=b=k=\sqrt [4] {\dfrac{4}{7}}$
[RIGHT][I][B]Nguồn: MathScope.ORG[/B][/I][/RIGHT]
 
__________________
Tú Văn Ninh
JokerNVT is offline   Trả Lời Với Trích Dẫn
Old 22-07-2012, 06:25 PM   #21
ptk_1411
Moderator
 
ptk_1411's Avatar
 
Tham gia ngày: Apr 2011
Bài gởi: 698
Thanks: 162
Thanked 813 Times in 365 Posts
Trích:
Nguyên văn bởi JokerNVT View Post
Mình có 1 thắc mắc nhỏ là sao bạn đoán được điểm rơi của bài này là $a=b=k=\sqrt [4] {\dfrac{4}{7}}$
Trước hết do biểu thức là đối xứng nên ta đoán dấu bằng xảy ra khi $a=b$. Viết lại biểu thức: $$P=\dfrac{2a}{(a^4+4)^2}$$

Dễ thấy cần AM-GM mẫu, nhưng là AM-GM mấy số để khử được tử? Để ý tử bậc 1, còn mẫu bậc 8 nên ta sẽ AM-GM 8 số. Công việc còn lại là dễ dàng: $$a^4+4=a^4+7.\dfrac{4}{7}\ge 8\sqrt[8]{a^4.\dfrac{4^7}{7^7}}$$

Dấu bằng xảy ra khi $a=\sqrt[4]{\dfrac{4}{7}}$.
[RIGHT][I][B]Nguồn: MathScope.ORG[/B][/I][/RIGHT]
 
__________________
P.T.K
Có xa xôi mấy mà tình xa xôi...
ptk_1411 is offline   Trả Lời Với Trích Dẫn
The Following 10 Users Say Thank You to ptk_1411 For This Useful Post:
1110004 (11-06-2013), Akira Vinh HD (22-07-2012), hgly1996 (26-07-2012), Hmh1996 (20-11-2012), hoang_kkk (22-07-2012), JokerNVT (22-07-2012), Lê Đình Mẫn (23-07-2012), TNP (25-07-2012), vuive97 (09-08-2012), zớt (22-07-2012)
Old 22-07-2012, 09:39 PM   #22
Trầm
+Thành Viên Danh Dự+
 
Tham gia ngày: Feb 2011
Bài gởi: 657
Thanks: 388
Thanked 470 Times in 196 Posts
Trích:
Nguyên văn bởi ttptcva3 View Post
Mình làm cách biến đổi như sau:
Từ điều kiện ta cm được: a+b+c$\le 3 $
Từ điều kiện bạn sẽ chứng minh được $a+b+c \ge 3$
[RIGHT][I][B]Nguồn: MathScope.ORG[/B][/I][/RIGHT]
 
__________________
Trầm is offline   Trả Lời Với Trích Dẫn
Old 23-07-2012, 02:20 PM   #23
Snow Bell
+Thành Viên Danh Dự+
 
Tham gia ngày: Apr 2012
Đến từ: Heaven
Bài gởi: 579
Thanks: 10
Thanked 513 Times in 283 Posts
Trích:
Nguyên văn bởi pexea12 View Post
Bài 7 Cho $a,b,c>0$ và $a+b+c=3$. Chứng minh rằng
$$\dfrac{1}{1+ab}+\dfrac{1}{1+bc}+\dfrac{1}{1+ca} \ge \dfrac{9}{2(\sqrt{a}+\sqrt{b}+\sqrt{c})}$$
Trích:
Nguyên văn bởi Ispectorgadget View Post
TA có:
$$\frac{1}{1+ab}=1-\frac{ab}{1+ab}\geq 1-\frac{\sqrt{ab}}{2}$$
$$\frac{1}{1+ab}+\frac{1}{1+bc}+\frac{1}{1+ac}\geq 3-\frac{\sqrt{ab}+\sqrt{ac}+\sqrt{bc}}{2}=3-\frac{(\sqrt{a}+\sqrt{b}+\sqrt{c})^2-a-b-c}{4}$$
Ta cần chứng minh: $$\frac{15}{4}-\frac{(\sum \sqrt{a})^2}{4}\geq \frac{9}{2(\sum \sqrt{a})}$$
Đặt $t=\sqrt{a}+\sqrt{b}+\sqrt{c}$
Để ý rằng $$\sqrt{a+b+c}\leq \sqrt{a}+\sqrt{b}+\sqrt{c}\leq \sqrt{3(a+b+c)}$$
Nên $\Rightarrow \sqrt{3}<t\leq 3$ bất đẳng thức được viết lại thành
$$\frac{15}{4}-\frac{t^2}{4}\geq \frac{9}{2t}\Leftrightarrow (t-3)(t-\frac{\sqrt{33}-\sqrt{3}}{2})(t+\frac{\sqrt{33}-\sqrt{3}}{2})\leq 0$$
Dễ thấy $\sqrt{3}>\frac{\sqrt{33}-\sqrt{3}}{2}$ nên đánh giá đúng với $t \in (\sqrt{3};3]$
Phép chứng minh hoàn tất $\blacksquare$
Bài toán này cũng có thể làm như sau.Bất đẳng thức cần chứng minh tương đương:
$$ \frac{ab}{1+ab}+\frac{bc}{1+bc}+\frac{ca}{1+ca} \le 3-\frac{9}{2(\sqrt{a}+\sqrt{b}+\sqrt{c})} $$
Áp dụng bất đẳng thức AM-GM:
$$ \frac{ab}{1+ab}+\frac{bc}{1+bc}+\frac{ca}{1+ca} \le \frac{1}{2}(\sqrt{ab}+\sqrt{bc}+\sqrt{ca}) $$
Như vậy bài toán quay về chứng minh:
$$ \sqrt{ab}+\sqrt{bc}+\sqrt{ca}+\frac{9}{\sqrt{a}+ \sqrt{b}+\sqrt{c}} \le 6 $$
Viết lại bất đẳng thức dưới dạng thuần nhất:
$$ \sqrt{ab}+\sqrt{bc}+\sqrt{ca}+\frac{(a+b+c)\sqrt{3 (a+b+c)}}{\sqrt{a}+\sqrt{b}+\sqrt{c}} \le 2(a+b+c) $$
Chuẩn hóa $ \sqrt{a}+\sqrt{b}+\sqrt{c}=1 \Rightarrow a+b+c=1-2(\sqrt{ab}+\sqrt{bc}+\sqrt{ca}) $
Vậy ta phải chứng minh:
$$ \sqrt{ab}+\sqrt{bc}+\sqrt{ca}+1-2(\sqrt{ab}+\sqrt{bc}+\sqrt{ca})\sqrt{3[1-2(\sqrt{ab}+\sqrt{bc}+\sqrt{ca})]} \le 2[1-2(\sqrt{ab}+\sqrt{bc}+\sqrt{ca})] $$
Tương đương:
$$ 3[1-2(\sqrt{ab}+\sqrt{bc}+\sqrt{ca})]^3 \le [(2-5(\sqrt{ab}+\sqrt{bc}+\sqrt{ca})]^2 $$
Hay:
$$ 0 \le [1-3(\sqrt{ab}+\sqrt{bc}+\sqrt{ca})][-8(\sqrt{ab}+\sqrt{bc}+\sqrt{ca})^2+\sqrt{ab}+\sqrt {bc}+\sqrt{ca}+1] $$
Điều này hiển nhiên đúng do $ \sqrt{ab}+\sqrt{bc}+\sqrt{ca} \le \dfrac{1}{3} $
[RIGHT][I][B]Nguồn: MathScope.ORG[/B][/I][/RIGHT]
 

thay đổi nội dung bởi: Snow Bell, 24-07-2012 lúc 12:29 AM
Snow Bell is offline   Trả Lời Với Trích Dẫn
The Following User Says Thank You to Snow Bell For This Useful Post:
TrauBo (26-07-2012)
Old 25-07-2012, 03:22 AM   #24
pexea12
+Thành Viên+
 
Tham gia ngày: Jul 2010
Đến từ: HUS
Bài gởi: 81
Thanks: 58
Thanked 56 Times in 35 Posts
Trích:
Nguyên văn bởi Ispectorgadget View Post
Bài 9: CMR nếu $a,b\in (0;1]$ thì $$a^{b-a}+b^{a-b}\leq 2$$


Không mất tính tổng quát, giả sử $0 < b \le a \le 1$

Áp dụng bất đẳng thức Bernoulli: Vì $0 \le a-b \le 1$ nên $b^{a-b}=(b-1+1)^{a-b} \le 1+(b-1)(a-b)$

$$\Rightarrow a^{b-a}+b^{a-b} \le 1+(b-1)(a-b)+a^{b-a}$$

Do đó ta cần chứng minh: $$a^{b-a}+(b-1)(a-b) \le 1$$

Đặt $t=b-a$ ($-1 \le t \le 0$) thì bất đẳng thức cần chứng minh tương đương với: $a^t-t(a+t-1) \le 1$ (1)

Xét hàm số $f(t)=a^t-t(a+t-1) \Rightarrow f'(t)=t \ln a - a-2t+1$

Phương trình $f'(t)=0$ có nghiệm $t=\dfrac{a-1}{\ln a -2} \ge 0$

Do đó với $-1 \le t \le 0$ thì $f'(t) \ge 0 \Rightarrow f(t)$ nghịch biến.

$\Rightarrow f(t) \le f(0)=1 \Rightarrow a^t-t(a+t-1) \le 1$ ((1) đúng) (đpcm)
[RIGHT][I][B]Nguồn: MathScope.ORG[/B][/I][/RIGHT]
 
__________________
"I don't quit once I step on court"

thay đổi nội dung bởi: pexea12, 25-07-2012 lúc 08:35 PM
pexea12 is offline   Trả Lời Với Trích Dẫn
Old 25-07-2012, 10:54 AM   #25
vjpd3pz41iuai
+Thành Viên+
 
vjpd3pz41iuai's Avatar
 
Tham gia ngày: Dec 2011
Bài gởi: 303
Thanks: 129
Thanked 130 Times in 81 Posts
Bài 10:Cho $a,b,c> 0 $.CMR
$$\dfrac{a^{3}c}{1+c(a^{2}+b^{2})}+d\frac{b^{3}a}{ 1+a(b^{2}+c^{2})}+ \dfrac{c^{3}b}{1+b(c^{2}+a^{2})}\geq \dfrac{abc(a+b+c)}{1+2abc}$$
[RIGHT][I][B]Nguồn: MathScope.ORG[/B][/I][/RIGHT]
 
__________________

thay đổi nội dung bởi: ptk_1411, 25-07-2012 lúc 10:33 PM Lý do: Đặt công thức vào thẻ $$ $$
vjpd3pz41iuai is offline   Trả Lời Với Trích Dẫn
Old 25-07-2012, 07:03 PM   #26
JokerNVT
+Thành Viên Danh Dự+
 
JokerNVT's Avatar
 
Tham gia ngày: Dec 2011
Đến từ: Trần Đại Nghĩa high school
Bài gởi: 571
Thanks: 206
Thanked 355 Times in 241 Posts
Trích:
Nguyên văn bởi vjpd3pz41iuai View Post
Bài 10:Cho $a,b,c> 0 $.CMR
$\frac{a^{3}c}{1+c(a^{2}+b^{2})}+\frac{b^{3}a}{1+a( b^{2}+c^{2})}+\frac{c^{3}b}{1+b(c^{2}+a^{2})}\geq \frac{abc(a+b+c)}{1+2abc} $
Biến đổi bất đẳng thức theo phương pháp Cauchy ngược dấu:
$$VT=\sum \dfrac{a^3c+ab^2c+a}{1+c(a^2+b^2)}=a+b+c-\sum \dfrac{a}{1+c(a^2+b^2)}-\sum \dfrac{ab^2c}{1+c(a^2+b^2)}$$
Áp dụng Cauchy:
$$a^2+b^2\ge 2ab \Rightarrow c(a^2+b^2)\ge 2abc$$
$$\Rightarrow VT\ge a+b+c-\dfrac{a+b+c}{1+2abc}-\dfrac{abc(a+b+c)}{1+2abc}$$
Mặt khác ta có:
$$a+b+c=\dfrac{(a+b+c)(1+2abc)}{1+2abc}=\dfrac{(a+ b+c)}{1+2abc}+\dfrac{2abc(a+b+c)}{1+2abc}$$
$$\Rightarrow a+b+c-\dfrac{a+b+c}{1+2abc}-\dfrac{abc(a+b+c)}{1+2abc}=\dfrac{abc(a+b+c)}{1+2a bc}$$
Vậy ta có đpcm
[RIGHT][I][B]Nguồn: MathScope.ORG[/B][/I][/RIGHT]
 
__________________
Tú Văn Ninh
JokerNVT is offline   Trả Lời Với Trích Dẫn
Old 25-07-2012, 07:19 PM   #27
vjpd3pz41iuai
+Thành Viên+
 
vjpd3pz41iuai's Avatar
 
Tham gia ngày: Dec 2011
Bài gởi: 303
Thanks: 129
Thanked 130 Times in 81 Posts
Bài 11:Cho $ x,y,z>0 $.Tìm GTLN của biểu thức:
$$P=\dfrac{x+y}{\sqrt[3]{2x^{3}+6y^{3}+xy(y-x)}+\sqrt[3]{2y^{3}+6x^{3}+xy(x-y)}}$$
[RIGHT][I][B]Nguồn: MathScope.ORG[/B][/I][/RIGHT]
 
__________________

thay đổi nội dung bởi: ptk_1411, 25-07-2012 lúc 10:33 PM
vjpd3pz41iuai is offline   Trả Lời Với Trích Dẫn
Old 25-07-2012, 10:33 PM   #28
bboy114crew
+Thành Viên+
 
Tham gia ngày: Oct 2010
Đến từ: Dòng thời gian...
Bài gởi: 294
Thanks: 290
Thanked 189 Times in 91 Posts
Gửi tin nhắn qua Yahoo chát tới bboy114crew
Trích:
Nguyên văn bởi vjpd3pz41iuai View Post
Bài 11:Cho $ x,y,z>0 $.Tìm GTLN của biểu thức:
$P=\frac{x+y}{\sqrt[3]{2x^{3}+6y^{3}+xy(y-x)}+\sqrt[3]{2y^{3}+6x^{3}+xy(x-y)}} $
Ta có các đánh giá sau thì bài này xong;
$2a^3+b^3 \ge 3a^2b$
$2a^2+ab+5b^2 \ge (2a+b)^2$
Với $a,b \ge 0$

Bài tiếp này:
Bài 12:
Cho các số thực dương $x,y,z $ thỏa mãn: $x+y+z=1$.Tìm giá trị nhỏ nhất của biểu thức:
$A=\dfrac{1}{xyz}-54(x^3+y^3+z^3)-(\dfrac{x}{yz}+\dfrac{y}{xz}+\dfrac{z}{yx})$
[RIGHT][I][B]Nguồn: MathScope.ORG[/B][/I][/RIGHT]
 

thay đổi nội dung bởi: Trầm, 25-07-2012 lúc 10:50 PM Lý do: Đánh số bài.
bboy114crew is offline   Trả Lời Với Trích Dẫn
Old 25-07-2012, 11:20 PM   #29
vjpd3pz41iuai
+Thành Viên+
 
vjpd3pz41iuai's Avatar
 
Tham gia ngày: Dec 2011
Bài gởi: 303
Thanks: 129
Thanked 130 Times in 81 Posts
Trích:
Nguyên văn bởi bboy114crew View Post
Ta có các đánh giá sau thì bài này xong;
$2a^3+b^3 \ge 3a^2b$
$2a^2+ab+5b^2 \ge (2a+b)^2$
Với $a,b \ge 0$
Bạn làm rõ hơn được không?Cách này không giống cách mìnhVới cả đánh giá thứ 2 sai rồi
[RIGHT][I][B]Nguồn: MathScope.ORG[/B][/I][/RIGHT]
 
__________________

thay đổi nội dung bởi: vjpd3pz41iuai, 25-07-2012 lúc 11:32 PM
vjpd3pz41iuai is offline   Trả Lời Với Trích Dẫn
Old 26-07-2012, 09:20 AM   #30
hoangduyenkhtn
+Thành Viên+
 
Tham gia ngày: Dec 2008
Bài gởi: 71
Thanks: 56
Thanked 57 Times in 36 Posts
Mình muốn trao đổi thêm một chút về bài toán 10. Bài toán thực chất là bài toán sau. Để thấy điều này bạn chia cả 2 vế cho abc
Cho a,b,c là các số thực dương, m là số thực không âm. Khi đó ta có:
$\frac{a^3}{a^2+b^2+mab}+\frac{b^3}{b^2+c^2+mbc}+
\frac{c^3}{c^2+a^2+mca}\ge\frac{a+b+c}{2+m} $
Bài toán này chứng minh bằng AM-GM ngược dấu.
Bài toán trên thực chất là $m=\frac{1}{abc} $
Bạn có thể thay đổi tùy ý m để được các bất đẳng thức khác và có thể sáng tạo ra nhiều bất đẳng thức khác cho dạng toán này và các bạn có thể tham khảo thêm ở các bài giảng về bất đẳng thức côsi của thầy nguyễn vũ lương.Chúc toàn thể diễn đàn ngày mới vui vẻ và làm việc hiệu quả.
[RIGHT][I][B]Nguồn: MathScope.ORG[/B][/I][/RIGHT]
 

thay đổi nội dung bởi: hoangduyenkhtn, 26-07-2012 lúc 09:25 AM Lý do: bổ sung thêm
hoangduyenkhtn is offline   Trả Lời Với Trích Dẫn
The Following User Says Thank You to hoangduyenkhtn For This Useful Post:
trandaiduongbg (16-05-2014)
Trả lời Gởi Ðề Tài Mới

Bookmarks

Ðiều Chỉnh
Xếp Bài

Quuyền Hạn Của Bạn
You may not post new threads
You may not post replies
You may not post attachments
You may not edit your posts

BB code is Mở
Smilies đang Mở
[IMG] đang Mở
HTML đang Tắt

Chuyển đến


Múi giờ GMT. Hiện tại là 03:57 PM.


Powered by: vBulletin Copyright ©2000-2024, Jelsoft Enterprises Ltd.
Inactive Reminders By mathscope.org
[page compression: 110.42 k/127.15 k (13.16%)]